Happy Memorial Day! Please note that AoPS Online is closed May 24-26th.

G
Topic
First Poster
Last Poster
Point satisfies triple property
62861   36
N an hour ago by cursed_tangent1434
Source: USA Winter Team Selection Test #2 for IMO 2018, Problem 2
Let $ABCD$ be a convex cyclic quadrilateral which is not a kite, but whose diagonals are perpendicular and meet at $H$. Denote by $M$ and $N$ the midpoints of $\overline{BC}$ and $\overline{CD}$. Rays $MH$ and $NH$ meet $\overline{AD}$ and $\overline{AB}$ at $S$ and $T$, respectively. Prove that there exists a point $E$, lying outside quadrilateral $ABCD$, such that
[list]
[*] ray $EH$ bisects both angles $\angle BES$, $\angle TED$, and
[*] $\angle BEN = \angle MED$.
[/list]

Proposed by Evan Chen
36 replies
62861
Jan 22, 2018
cursed_tangent1434
an hour ago
Inspired by 2025 Beijing
sqing   4
N an hour ago by pooh123
Source: Own
Let $ a,b,c,d >0  $ and $ (a^2+b^2+c^2)(b^2+c^2+d^2)=36. $ Prove that
$$ab^2c^2d \leq 8$$$$a^2bcd^2 \leq 16$$$$ ab^3c^3d \leq \frac{2187}{128}$$$$ a^3bcd^3 \leq \frac{2187}{32}$$
4 replies
sqing
Yesterday at 4:56 PM
pooh123
an hour ago
Find the minimum
sqing   27
N an hour ago by sqing
Source: Zhangyanzong
Let $a,b$ be positive real numbers such that $a^2b^2+\frac{4a}{a+b}=4.$ Find the minimum value of $a+2b.$
27 replies
sqing
Sep 4, 2018
sqing
an hour ago
Sharygin 2025 CR P15
Gengar_in_Galar   7
N an hour ago by Giant_PT
Source: Sharygin 2025
A point $C$ lies on the bisector of an acute angle with vertex $S$. Let $P$, $Q$ be the projections of $C$ to the sidelines of the angle. The circle centered at $C$ with radius $PQ$ meets the sidelines at points $A$ and $B$ such that $SA\ne SB$. Prove that the circle with center $A$ touching $SB$ and the circle with center $B$ touching $SA$ are tangent.
Proposed by: A.Zaslavsky
7 replies
Gengar_in_Galar
Mar 10, 2025
Giant_PT
an hour ago
Inequality olympiad algebra
Foxellar   1
N an hour ago by sqing
Given that \( a, b, c \) are nonzero real numbers such that
\[
\frac{1}{abc} + \frac{1}{a} + \frac{1}{c} = \frac{1}{b},
\]let \( M \) be the maximum value of the expression
\[
\frac{4}{a^2 + 1} + \frac{4}{b^2 + 1} + \frac{7}{c^2 + 1}.
\]Determine the sum of the numerator and denominator of the simplified fraction representing \( M \).
1 reply
Foxellar
6 hours ago
sqing
an hour ago
Inspired by RMO 2006
sqing   2
N an hour ago by sqing
Source: Own
Let $ a,b >0  . $ Prove that
$$  \frac {a^{2}+1}{b+k}+\frac { b^{2}+1}{ka+1}+\frac {2}{a+kb}  \geq \frac {6}{k+1}  $$Where $k\geq 0.03 $
$$  \frac {a^{2}+1}{b+1}+\frac { b^{2}+1}{a+1}+\frac {2}{a+b}  \geq 3  $$
2 replies
sqing
Yesterday at 3:24 PM
sqing
an hour ago
Inspired by 2025 Xinjiang
sqing   3
N an hour ago by sqing
Source: Own
Let $ a,b >0  . $ Prove that
$$  \left(1+\frac {a} { b}\right)\left(6+\frac {b}{a}\right) \left(2+\frac {a}{b}+\frac {b}{ a}\right)  \geq\frac {625}{ 12}$$$$  \left(1+\frac {a} { b}\right)\left(6+\frac {a}{b}\right) \left(2+\frac {a}{b}+\frac {b}{ a}\right)  \geq29+6\sqrt 6$$$$  \left(1+\frac {a} { b}\right)\left(2+\frac {b}{ a}\right) \left(2+\frac {a}{b}+\frac {b}{ a}\right)  \geq \frac{3(63+11\sqrt{33})}{16}  $$$$  \left(1+\frac {a} { b}\right)\left(2+\frac {a}{ b}\right) \left(2+\frac {a}{b}+\frac {b}{ a}\right)  \geq \frac{223+70\sqrt{10}}{27}  $$
3 replies
sqing
Yesterday at 5:56 PM
sqing
an hour ago
interesting diophantiic fe in natural numbers
skellyrah   2
N 2 hours ago by SYBARUPEMULA
Find all functions \( f : \mathbb{N} \to \mathbb{N} \) such that for all \( m, n \in \mathbb{N} \),
\[
mn + f(n!) = f(f(n))! + n \cdot \gcd(f(m), m!).
\]
2 replies
skellyrah
Yesterday at 8:01 AM
SYBARUPEMULA
2 hours ago
PHP on subsets
SYBARUPEMULA   0
2 hours ago
Source: inspired by Romania 2009
Let $X$ be a set of $102$ elements. Let $A_1, A_2, A_3, ..., A_{101}$ be subsets of $X$ such that the union of any $50$ of them has more than $100$ elements. Show that there's a member of $X$ that occurs in at least $7$ different subsets $A_j$.
0 replies
SYBARUPEMULA
2 hours ago
0 replies
Moscow Geometry Problems
nataliaonline75   1
N 2 hours ago by MathLuis
Source: MMO 2003 10.4
Let M be the intersection point of the medians of ABC. On the perpendiculars dropped from M to sides BC, AC, AB, points A1, B1, C1 are taken, respectively, with A1B1 perpendicular to MC and A1C1 perpendicular to MB. prove that M is the intersections pf the medians in A1B1C1.
Any solutions without vectors? :)
1 reply
nataliaonline75
Jul 9, 2024
MathLuis
2 hours ago
Serbian selection contest for the IMO 2025 - P4
OgnjenTesic   1
N 3 hours ago by nataliaonline75
Source: Serbian selection contest for the IMO 2025
For a permutation $\pi$ of the set $A = \{1, 2, \ldots, 2025\}$, define its colorfulness as the greatest natural number $k$ such that:
- For all $1 \le i, j \le 2025$, $i \ne j$, if $|i - j| < k$, then $|\pi(i) - \pi(j)| \ge k$.
What is the maximum possible colorfulness of a permutation of the set $A$? Determine how many such permutations have maximal colorfulness.

Proposed by Pavle Martinović
1 reply
OgnjenTesic
May 22, 2025
nataliaonline75
3 hours ago
T lies on Euler circle
ken3k06   6
N Jun 1, 2022 by jayme
Let $\displaystyle ABC$ be a triangle with altitudes $\displaystyle AD,BE,CF$ intersect at orthocenter $\displaystyle H$. Let $\displaystyle T$ be an arbitrary point lies on $\displaystyle ( DEF)$. $\displaystyle D'$ be the reflection point of $\displaystyle D$ on $\displaystyle AT$. $\displaystyle D'T$ intersects $\displaystyle EF$ at $\displaystyle P$. Prove that $\displaystyle ( DTP)$ passes through the circumcenter of $\displaystyle ADP$.

IMAGE
6 replies
ken3k06
May 25, 2022
jayme
Jun 1, 2022
T lies on Euler circle
G H J
G H BBookmark kLocked kLocked NReply
The post below has been deleted. Click to close.
This post has been deleted. Click here to see post.
ken3k06
424 posts
#1
Y by
Let $\displaystyle ABC$ be a triangle with altitudes $\displaystyle AD,BE,CF$ intersect at orthocenter $\displaystyle H$. Let $\displaystyle T$ be an arbitrary point lies on $\displaystyle ( DEF)$. $\displaystyle D'$ be the reflection point of $\displaystyle D$ on $\displaystyle AT$. $\displaystyle D'T$ intersects $\displaystyle EF$ at $\displaystyle P$. Prove that $\displaystyle ( DTP)$ passes through the circumcenter of $\displaystyle ADP$.

https://scontent.fdad3-1.fna.fbcdn.net/v/t1.15752-9/280650918_1427756197694764_7145749033231498711_n.png?_nc_cat=103&ccb=1-7&_nc_sid=ae9488&_nc_ohc=FnjTOESeskgAX_1wK43&_nc_ht=scontent.fdad3-1.fna&oh=03_AVLcP-KSAuX1yT9QQnr-7uA5KQKqpYNK3FAoMz3bkSIasQ&oe=62B1F047
This post has been edited 1 time. Last edited by ken3k06, May 25, 2022, 1:38 AM
Z K Y
The post below has been deleted. Click to close.
This post has been deleted. Click here to see post.
jayme
9801 posts
#2 • 1 Y
Y by ken3k06
Any ideas?

Sincerely
Jean-Louis
Z K Y
The post below has been deleted. Click to close.
This post has been deleted. Click here to see post.
VZH
60 posts
#3 • 1 Y
Y by ken3k06
Hints:
We only need to prove $\angle PTD= 2\angle PAD$, and because $D$ and $D'$ are symmetric wrt $AT$, it suffices to show $(ATP)$ is tangent to $AD$.
Consider an inversion wrt $A$ that swaps $(B,F)$ and $(D,E)$. Now, this inversion sends $P, T$ to $P', T'$, where $P'$ and $T'$ lie on $(ABC)$ and $(BHC)$ respectively. Let $H'$ be the reflection of $H$ on $AT$, then the inversion also swaps $(H',D')$, and so $A,T',G',H'$ are concyclic. From here, proving $T'G' \parallel AD$ is easy.
Z K Y
The post below has been deleted. Click to close.
This post has been deleted. Click here to see post.
jayme
9801 posts
#4 • 2 Y
Y by ken3k06, Mango247
Dear,
thank you for your proof...
Do you have an idea for a synthetic proof without inversion?

Sincerely
Jean-Louis
Z K Y
The post below has been deleted. Click to close.
This post has been deleted. Click here to see post.
jayme
9801 posts
#5 • 1 Y
Y by ken3k06
Bump!

Sincerely
Jean-Louis
Z K Y
The post below has been deleted. Click to close.
This post has been deleted. Click here to see post.
jayme
9801 posts
#6 • 1 Y
Y by ken3k06
last Bump!

Sincerely
Jean-Louis
Z K Y
The post below has been deleted. Click to close.
This post has been deleted. Click here to see post.
jayme
9801 posts
#7
Y by
Dear Mathlinkersn
finally I have found a proof...

Any references?

Thank in advance
Sincerely
Jean-Louis
Z K Y
N Quick Reply
G
H
=
a